Multivariable derivatives problem?

Click For Summary
The discussion revolves around finding the multivariable derivative f_{xyz} for the function f(x,y,z) = u(t), where t = xyz. Participants clarify that although the problem appears complex, it essentially requires applying the chain rule for a single-variable function. The derivatives are expressed in terms of u'(t), u''(t), and u'''(t), leading to the conclusion that f_{xyz} can be rewritten using t. Ultimately, the key takeaway is that substituting "xyz" with "t" simplifies the expression for f_{xyz}. Understanding this relationship is crucial for solving the problem effectively.
Jormungandr
Messages
30
Reaction score
0

Homework Statement



Let f(x,y,z)=u(t), where t=xyz. Show that f_{xyz} = F(t) and find F(t).

The Attempt at a Solution



I'm a little confused about the presentation of the variables in this problem. What does F(t) refer to? This isn't a chain rule question, because it's presented before chain rule is introduced. I'm just not sure how to go about finding each partial derivative since u(t) isn't explicitly given... Some advice would be appreciated!
 
Physics news on Phys.org
It is a chain rule problem, but uses the chain rule for a function of a single variable that you saw back in first semester Calculus. Saying "f(x,y,z)=u(t), where t=xyz" means that f is really as single function of the product xyz rather than a more general function of x, y, and z. For example, f(x, y, z) might be "xyz+ 1" or it might be "(xyz)^3" or "sin(xyz)" but cannot be "x+ 3y+ 2z" or "x^2+yz".

If f(x, y, z)= u(t) with t= xyz, then f_x= u'(xyz)(yz), f_y= u'(xyz)(xz), and f_z= u'(xyz)(xy).

with f_x= u'(xyz)yz, then f_{xy}= u''(xyz)(xz)(yz)+ u'(xyz)z= u''(xyz)xyz^2+ u'(xyz)z and f_{xyz)= u'''(xyz)(xy)(xyz^2)+ 2u''(xyz)xyz+ u''(xyz)xyz+ u'(xyz)=
u'''(xyz)x^2y^2z^2+ 3u''(xyz)xyz)+ u'(xyz).

Now, what do you get if you replace every "xyz" in that with t?
 
Question: A clock's minute hand has length 4 and its hour hand has length 3. What is the distance between the tips at the moment when it is increasing most rapidly?(Putnam Exam Question) Answer: Making assumption that both the hands moves at constant angular velocities, the answer is ## \sqrt{7} .## But don't you think this assumption is somewhat doubtful and wrong?

Similar threads

Replies
4
Views
1K
Replies
2
Views
1K
Replies
10
Views
3K
  • · Replies 12 ·
Replies
12
Views
2K
  • · Replies 3 ·
Replies
3
Views
3K
Replies
7
Views
2K
  • · Replies 13 ·
Replies
13
Views
1K
  • · Replies 2 ·
Replies
2
Views
2K
  • · Replies 4 ·
Replies
4
Views
2K
  • · Replies 3 ·
Replies
3
Views
2K